LSAT and Law School Admissions Forum

Get expert LSAT preparation and law school admissions advice from PowerScore Test Preparation.

 Erik Shum
PowerScore Staff
  • PowerScore Staff
  • Posts: 25
  • Joined: Jul 25, 2019
|
#66902
Hi Snowy,

In regards to answer choice (A):
"Any time" should not be the focus of your analysis, in and of itself. If the answer choice were modified to say "any time a medical procedure results in the patient's death, the medical team is guilty of manslaughter", it would be a correct answer choice. "Any time" is not an obstacle to justify the stimulus's conclusion, even if it is unnecessarily broad. To justify a conclusion, it does not hurt if the answer choice does more than is necessary (as Adam Tyson's earlier post pointed out, the correct answer also does more than is necessary: "and only if" could be omitted and answer choice (D) would remain correct).

And in regards to "could be" charged, similar to what James Finch posted earlier, that part of answer choice (A) fails to match the stimulus's conclusion that the medical team is technically guilty so you would have to be certain that being charged and being technically guilty are the same thing (and, of course, they are not).

In regards to answer choice (B):
I am not sure I would describe it as being "too broad" but it looks like you had the correct focus: Even if the statement in answer choice (B) were true, it might not apply to the procedure described in the stimulus.

I hope that answers your question!
 dlehr99
  • Posts: 24
  • Joined: Dec 06, 2019
|
#72797
Adam Tyson wrote: So, if you deliberately stop the life functions and the patient dies, then you are guilty of manslaughter - that is what we need to strengthen.

Look for an answer that makes "guilty of manslaughter" necessary and makes "deliberately stop the life functions" and "functions do not return" sufficient, and you'll have made the link you needed to make.
I'm not sure how to know that:

Because the doctors deliberately stop...
AND
If these functions do not resume... should be combined by "AND" as was explained.

I saw one conditional - if these functions do not resume.... then the medical team is guilty of manslaughter. But I don't follow how, because the doctors deliberately... fits into this. I recognized that it should but the process felt unfamiliar.

Also could you please confirm that in this question type we are simply trying to mirror the conditional relationship that was stated. So we need a sufficient and necessary condition, respectively that matches to the stimulus?
 Paul Marsh
PowerScore Staff
  • PowerScore Staff
  • Posts: 290
  • Joined: Oct 15, 2019
|
#72866
Hi dlehr99! The conclusion here says that the medical team would be guilty of manslaughter. The concept of being guilty of manslaughter doesn't appear anywhere in the rest of the stimulus. Whenever we have a new concept in the conclusion that isn't really explained by the premises, that is a clear gap in the argument.

For example, take this simple argument: "Jeremy wants a stylish car. Therefore Jeremy should buy a silver car." The concept of "silver" is new in that conclusion (in other words it wasn't present in the premises), and so there is a "gap" between the conclusion and the premises, since logically a conclusion can only come from what the premises provide. A strengthener should smooth over the "gap" in an argument, so we want our strengthener to connect the premises of the argument to the new concept in the conclusion. So a good strengthener for that argument would connect the premises with the new "silver" concept from the conclusion, with something like: "All stylish cars are silver".

Similarly, here the presence of "guilty of manslaughter" in the conclusion is a clear gap as it wasn't explained by the premises. We want a strengthener to smooth the gap by tying together the premises of the argument and the new concept of "guilty of manslaughter" in the conclusion. The easiest way to think about this is with a conditional statement whose necessary condition contains "guilty of manslaughter", and whose sufficient condition contains something from the premises. Thus, the conditional could be any number of things. Here are a few examples:

Deliberately stop life functions :arrow: Guilty of manslaughter
Patient dies :arrow: Guilty of manslaughter
Perform risky surgical procedure :arrow: Guilty of manslaughter
Ever member of any medical team :arrow: Guilty of manslaughter
Perform procedure with patient's consent :arrow: Guilty of manslaughter

Obviously, some of those examples are absurd. But they would all be correct answer choices! They all connect our premises to the new concept in the conclusion, and therefore successfully strengthen the argument by smoothing over its gap. Answer Choice (D), too, connects the new concept in the conclusion to the premises - deliberately stopping life functions and permanent cessation (i.e. the patient dying). Did (D) need to touch on both deliberately stopping life functions AND permanent cessation in order to be correct? No, but it did and that's fine. The important point is that elements of the premises were in the sufficient condition and the new concept in the conclusion was in the necessary condition. That smooths over the gap by explaining the presence of "guilty of manslaughter" in the conclusion, and therefore strengthens the argument. None of the other answer choices did this (as others have touched on in this thread, Answer Choice (A) was close but it discussed "charged with manslaughter" instead of "guilty of manslaughter"). Hope that helps!
 dlehr99
  • Posts: 24
  • Joined: Dec 06, 2019
|
#72924
Hi Paul,

Thanks a lot for that detailed explanation - it definitely cleared things up for me on this aspect.

So in this case it was a strengthen question and bringing in additional information (even more than we needed) was acceptable in the answer choice - A concept I'm familiar with. Had it been an assumption question that also would have been acceptable. However in a Justify question the extra would have excluded this from being the correct answer, right?

Generally I do pretty well in identifying what type between the three the test requires - this one has me confused though.

Could you possibly explain how if this question in particular were slightly different it would instead be an assumption and/or Justify?

Thanks again!
 James Finch
PowerScore Staff
  • PowerScore Staff
  • Posts: 943
  • Joined: Sep 06, 2017
|
#73020
Hi D. Lehr,

Assumptions, Justify and Strengthen answer choices all allow for new information to be brought. Essentially all three act as an extra premise that should make the argument stronger (and in the case of a correct Justify answer choice, logically valid) when paired up with the existing premise in the stimulus. Strengthen answers only strengthen the stimulus, and nothing else, while Assumption answers are necessary to the stimulus, which could not be true if the assumption isn't true, and Justify answers prove the stimulus's argument correct. All three act as an extra premise that brings in new information not stated in the stimulus, as do Weaken and Resolve the Paradox answers.

Hope this clears things up!
 danimcca
  • Posts: 9
  • Joined: Sep 06, 2019
|
#78948
In short, the stimulus needs:

Deliberately cause life functions to cease permanently :arrow: Guilty of Manslaughter, (answer choice (D))

while (B) gives us:

May Be Guilty of Manslaughter :arrow: Patient Dies

Hope this clears things up![/quote]

Hi can you go further into the explaining the whole conditional statement of B
I feel like I want to read it as : High risk of death --> death / manslaughter
 Jeremy Press
PowerScore Staff
  • PowerScore Staff
  • Posts: 1000
  • Joined: Jun 12, 2017
|
#78969
Hi danimcca,

Answer choice B is a tricky "nested conditional" sentence.

Initially we could diagram the first half of the sentence as a sufficient condition, since it's modified by "if," per below:
Very High Known Risk of Death :arrow:

Initially we could diagram the entire second half of the sentence as the necessary condition, since it's modified by "then," per below:
Very High Known Risk of Death :arrow: Only if patient dies can doctors be guilty of manslaughter


However, that diagram needs clarification, because that (long, wordy) necessary condition is its own conditional statement, that "only if" the patient dies can the doctor be guilty of manslaughter. This would diagram as:
Doctors Guilty of Manslaughter :arrow: Patient dies.

So a better/complete diagram of answer choice B would be:
Very High Risk of Death :arrow: (Doctors Guilty of Manslaughter :arrow: Patient Dies)

With nested conditionals, it's sometimes better to "translate" the rule into our own words. In this case, let's do that. What the sufficient condition is saying is that the rule covers certain circumstances, circumstances in which a procedure is known to carry a very high risk of death. And, in such circumstances, there's a possibility of a manslaughter charge for the doctors only if the patient dies.

This isn't a rule that goes as far as I'd like to help with justifying the conclusion for several fairly simple reasons:
1. The stimulus is making a conclusion about "risky surgical" procedures in general, whereas this rule in answer choice B is only talking about procedures for which there is a "known" and "very high risk of death." The rule thus cuts narrower than the stimulus argument.
2. The stimulus is making a conclusion about the manslaughter guilt of the entire "medical team," whereas the rule in answer choice B only talks about the manslaughter guilt of the "doctors" (only part of the team). This also narrows the rule in a way that makes it impossible to justify everything the conclusion discusses.
3. The stimulus conclusion is certain that the medical team is technically guilty of manslaughter. But the rule in answer choice B only gives us one of the things that's necessary for manslaughter guilt (the patient's death). Such a rule by its nature leaves open the possibility of other things that might be necessary for manslaughter guilt, and without knowing what those other things are (or whether there even are such things), we won't be able to know for sure that the medical team in the stimulus is technically guilty. {This is the "Mistaken Reversal" aspect of the rule that others point out that makes the rule too weak to help justify the conclusion.}

I hope this helps!

Get the most out of your LSAT Prep Plus subscription.

Analyze and track your performance with our Testing and Analytics Package.